Communities

Writing
Writing
Codidact Meta
Codidact Meta
The Great Outdoors
The Great Outdoors
Photography & Video
Photography & Video
Scientific Speculation
Scientific Speculation
Cooking
Cooking
Electrical Engineering
Electrical Engineering
Judaism
Judaism
Languages & Linguistics
Languages & Linguistics
Software Development
Software Development
Mathematics
Mathematics
Christianity
Christianity
Code Golf
Code Golf
Music
Music
Physics
Physics
Linux Systems
Linux Systems
Power Users
Power Users
Tabletop RPGs
Tabletop RPGs
Community Proposals
Community Proposals
tag:snake search within a tag
answers:0 unanswered questions
user:xxxx search by author id
score:0.5 posts with 0.5+ score
"snake oil" exact phrase
votes:4 posts with 4+ votes
created:<1w created < 1 week ago
post_type:xxxx type of post
Search help
Notifications
Mark all as read See all your notifications »
Q&A

Do the Faber partition polynomials have integer coefficients?

+3
−0

The Online Encyclopedia of Integer Sequences includes A263916: Coefficients of the Faber partition polynomials. Perhaps the clearest definition given is

-log(1 + b(1) x + b(2) x^2 + ...) = Sum_{n>=1} F(n,b(1),...,b(n)) * x^n/n

which in better notation is

$$\sum_{n \ge 1} F_n(b_1, \ldots, b_n) \frac{x^n}n = -\log(1 + b_1 x + b_2 x^2 + \cdots)$$

E.g. $$F_4(b_1,b_2,b_3,b_4) = -4b_4 + 4b_1 b_3 + 2b_2^2 - 4b_1^2 b_2 + b_1^4$$

The fact that it's listed in OEIS suggests that all of the coefficients should be integers, but none of the formulae or comments in the page obviously tells me that they are. What's the most straightforward way to see this?

History
Why does this post require moderator attention?
You might want to add some details to your flag.
Why should this post be closed?

0 comment threads

1 answer

+1
−0

Let $B(x) = b_1 x + b_2 x^2 + \cdots$. Then $$\begin{eqnarray*} F_n(b_1, \ldots, b_n) &=& - [x^n] n \log(1 + B(x)) \\ &=& [x^n] n \sum_{i \ge 1} \frac{(-B(x))^i}{i} \\ &=& \sum_{\lambda \, \vdash \, n} \frac{n}{\operatorname{len}(\lambda)} \binom{\operatorname{len}(\lambda)}{f_1, \ldots, f_n} \prod_j (-b_j)^{f_j} \ \end{eqnarray*}$$ where the sum in the last line is over partitions $\lambda = 1^{f_1}2^{f_2} \cdots n^{f_n}$ with $\sum_i if_i = n$ and the length $\operatorname{len}(\lambda)$ defined as $\sum_i f_i$.

So the question is whether $n \binom{\operatorname{len}(\lambda)}{f_1, \ldots, f_n}$ is divisible by $\operatorname{len}(\lambda)$. Consider a prime $p$ which divides $\operatorname{len}(\lambda)$ and look at $p$-adic valuations. Specifically, choose $c$ such that $\nu_p(f_c) = \min_i(\nu_p(f_i))$. We can split the multinomial as $$\binom{\operatorname{len}(\lambda)}{f_1, \ldots, f_n} = \binom{\operatorname{len}(\lambda)}{f_c} \binom{\operatorname{len}(\lambda) - f_c}{ \{ f_i : i \neq c \}}$$ By Kummer's theorem, $$\nu_p\left(\binom{\operatorname{len}(\lambda)}{f_c}\right) \ge \nu_p(\operatorname{len}(\lambda)) - \nu_p(f_c) \tag{1}$$

Since $\sum_i if_i = n$ we have $\nu_p(n) \ge \nu_p(f_c)$ and we can combine that with $(1)$ to get $$\nu_p\left(\binom{\operatorname{len}(\lambda)}{f_c}\right) + \nu_p(n) \ge \nu_p(\operatorname{len}(\lambda))$$ Therefore $$\nu_p\left( \frac{n}{\operatorname{len}(\lambda)} \binom{\operatorname{len}(\lambda)}{f_1, \ldots, f_n} \right) \ge 0$$ and since this holds for every prime divisor of the denominator, we have an integer.

History
Why does this post require moderator attention?
You might want to add some details to your flag.

0 comment threads

Sign up to answer this question »